Inscription / Connexion Nouveau Sujet
Niveau Maths sup
Partager :

Uniformement continue

Posté par
Rana
18-05-17 à 15:07

Bonjour , j'ai une question.
f(x)=\sqr{x} est elle uniformement continue sur]0;+[ ou sur [0;+[.

(Je sais comment la demontré mais juste ma question est pour les bornes).


Merci d'avance.

Posté par
lionel52
re : Uniformement continue 18-05-17 à 15:16

Elle est UC sur les 2 intervalles !

Posté par
Camélia Correcteur
re : Uniformement continue 18-05-17 à 15:16

Bonjour

Non, ni sur l'un ni sur l'autre, puisque |\sqrt x-\sqrt y|/|x-y| tend vers l'infini quand (x,y) tend vers (0,0) par valeurs positives.

Mais elle est uniformément continue sur [a,+\infty[ pour tout a > 0

Posté par
lionel52
re : Uniformement continue 18-05-17 à 15:19

Bonjour Camélia, ce que vous montrez c'est que f est non lipschitzienne, pas non UC!

Posté par
Rana
re : Uniformement continue 18-05-17 à 15:34

Okk merci lionel52

Posté par
Camélia Correcteur
re : Uniformement continue 18-05-17 à 15:48

En effet!

Posté par
etniopal
re : Uniformement continue 18-05-17 à 17:15

Pour une application f : X      et  t > 0 on  pose   w(f,t) = Sup [ |f(x) - f(y)| │  (x,y) X² et |x - y| < t } ( c'est un élément de [0 , +] )  .
t   w(f,t) est croissante  . On pose w(f) = Inf {w(f,t) | t > 0 }  ( ( c'est  aussi un élément de [0 , +] )  .

  On a :  f est  uniformément continue    SSI    w(f)  = 0 .

Soient a > 0  et   pa    l'application    continue de  +  vers   telle que  p a(x) = exp(aln(x)) si x > 0 . Le calcul de   w(pa)   se fait facilement .

Soit t > 0 .
On a      w( p[/sub],t) =  Sup{ p[sub]a(x + t)  - pa(x)  |  x   0   }   puisque pa  est croissante   .

  .si t  > 1 ,   x    pa(x + t)  - pa(x)   est  croissante  non bornée donc  pa  n'est pas UC .

  .si t 1  on a   w(pa,t) = ta  donc   pa  est UC

Posté par
jb2017
re : Uniformement continue 18-05-17 à 18:50

Bonjour  je suis d'accord avec etniopal.
Néanmoins on peut aussi voir l'uniforme continuité de cette fonction comme cela:
D'abord si il a y un problème d'uniforme continuité cela viendra de +\infty.
En effet la fonction étant continue sur [0,1],  elle est UC sur [0,1].
Maintenant sur [1,+\infty[  elle est k-lips (facile à voir avec le th des A.F) donc UC sur
[1,\infty[.
Cela implique donc l'UC sur R^+ =[0,1] \cup [1,+\infty[

Posté par
Rana
re : Uniformement continue 18-05-17 à 19:34

Merci

Moi je l'avait demontre comme suit en utilisant la definition de l'uniformement continue:

|f(x)-f(y)|=|\sqrt{x}-\sqrt{y}| =|\frac{x-y}{\sqrt{x}+\sqrt{y}}|\leq |\frac{x-y}{\sqrt{x}-\sqrt{y}}|\leq \frac{\delta }{|\sqrt{x}-\sqrt{y}|}

=> |\sqrt{x}-\sqrt{y}|^{2} \leq\delta  
=>|\sqrt{x}-\sqrt{y}|\leq\sqrt{\delta}
Alors >0 =\sqrt{\frac{\epsilon}{2}} telque x,y[0;+ ]on |x-y|<=> |f(x)-f(y)|<

Posté par
etniopal
re : Uniformement continue 18-05-17 à 22:26

1.
Tu n'as fait aucun raisonnement .

2.
Depuis quand  a-t- on : 1/(x + y)     1/|x  -  y|  pour tout x > 0 et tout y > 0           ?

Posté par
jb2017
re : Uniformement continue 18-05-17 à 23:49

Rebonjour
@etnopial,  pour la remarque 1  je suis d'accord.
D'ailleurs  à cause du problème en 0 , on devrait faire un travail particulier à ce niveau.
Je conseillerai à Rana d'imposer à \delta d'être inférieur à 1/2.
puis de considérer  2 cas
cas 1  x<=1  
cas 2  x>1   (ici x et y ne s'approchent pas de 0)

@etnopial pour la remarque 2, je pense que l'inégalité est vraie,non?  

Posté par
Rana
re : Uniformement continue 19-05-17 à 13:31

Rebonjour
Pour x 1, on a :
xx est continue sur [0;1] (compact) alors f(x)=x est uniformement continue

pour x>1 , on a:
|f(x)-f(y)=|x -y |

=| \frac{x-y}{\sqrt{x}+\sqrt{y}}|

Or x +y 2

\frac{1}{\sqrt{x}+\sqrt{y}}\frac{1}{2}

|f(x)-f(y)|\frac{\delta}{2}
Prenons alors =>0.
Donc >0 >0, x,y[1;+] , |x-y|<|f(x)-f(y)|<




Et pourquoi le raisonnement ici est faux?

Rana @ 18-05-2017 à 19:34

Merci

Moi je l'avait demontre comme suit en utilisant la definition de l'uniformement continue:

|f(x)-f(y)|=|\sqrt{x}-\sqrt{y}| =|\frac{x-y}{\sqrt{x}+\sqrt{y}}|\leq |\frac{x-y}{\sqrt{x}-\sqrt{y}}|\leq \frac{\delta }{|\sqrt{x}-\sqrt{y}|}

=> |\sqrt{x}-\sqrt{y}|^{2} \leq\delta  
=>|\sqrt{x}-\sqrt{y}|\leq\sqrt{\delta}
Alors >0 =\sqrt{\frac{\epsilon}{2}} telque x,y[0;+ ]on |x-y|<=> |f(x)-f(y)|<

Posté par
Oldboub
re : Uniformement continue 19-05-17 à 15:12

Bonjour,

"Depuis quand  a-t- on : 1/(Vx +V y)  <=   1/|Vx  -  Vy|  pour tout x > 0 et tout y > 0 ? "

A mon avis, si l'inégalité est vraie pour tout x, y > 0, si on prend x = y, il y aura un très gros problème....

Posté par
Rana
re : Uniformement continue 19-05-17 à 15:27

Bon ok, mais pourquoi vous avez considerer que ce n'était pas un raisonnement

etniopal @ 18-05-2017 à 22:26

1.
Tu n'as fait aucun raisonnement .



Et là
Rana @ 19-05-2017 à 13:31

Rebonjour
Pour x 1, on a :
xx est continue sur [0;1] (compact) alors f(x)=x est uniformement continue

pour x>1 , on a:
|f(x)-f(y)=|x -y |

=| \frac{x-y}{\sqrt{x}+\sqrt{y}}|

Or x +y 2

\frac{1}{\sqrt{x}+\sqrt{y}}\frac{1}{2}

|f(x)-f(y)|\frac{\delta}{2}
Prenons alors =>0.
Donc >0 >0, x,y[1;+] , |x-y|<|f(x)-f(y)|<



C'est juste ?

Posté par
ThierryPoma
re : Uniformement continue 19-05-17 à 16:23

Soit a, b\in\R^+ tels que a\leqslant{b}. Partant,

(\sqrt{b}-\sqrt{a})^2=b-2\,\sqrt{a\,b}+a

avec

0\leqslant{a}\leqslant{b}\Rightarrow{a^2}\leqslant{a\,b}\Rightarrow{a}\leqslant\sqrt{a\,b}\Rightarrow{b-2\,\sqrt{a\,b}+a}\leqslant{b-a}

(...)

L'idée est ici de montrer que la fonction racine carrée est 1/2-höldérienne sur \R^+.

Ne pas oublier que l'uniforme continuité est une propriété métrique, pas topologique.

Posté par
Oldboub
re : Uniformement continue 19-05-17 à 17:02

Rana,
Dans ton raisonnement, comment as tu fait pour faire apparaître .
Tu cherches à retomber sur la définition...Je pense que tu as considéré en avance que f était uniformément continue...donc ça coince pour ton choix d'epsilon...Donc t'as rien démontré.

Posté par
Rana
re : Uniformement continue 19-05-17 à 17:38

Oldboub @ 19-05-2017 à 17:02

Rana,
Dans ton raisonnement, comment as tu fait pour faire apparaître .
Tu cherches à retomber sur la définition...Je pense que tu as considéré en avance que f était uniformément continue...donc ça coince pour ton choix d'epsilon...Donc t'as rien démontré.


Moi ce que je comprend de cet definition

Rana @ 19-05-2017 à 13:31


>0 >0, x,y[1;+[ , |x-y|<|f(x)-f(y)|<


Qui est la definition de l'uniformement continue que pour tout epsilon il existerait un delta (qui dependra de epsilon) , de tel façon que si la distance entre x et y est plus petite qu'une valeur delta cela impliquera que la difference entre leur image sera plus petit qu'une valeur epsilon .
C'est pour cela à chaque fois que je vois |x-y|  je met (en considerant cela comme hypothese pour trouver un delta en fonction de epsilon, et je ne vois pas ce qu'il y a de faux Veillez m'eclaircir s'il vous plaît)

ThierryPoma @ 19-05-2017 à 16:23



0\leqslant{a}\leqslant{b}\Rightarrow{a^2}\leqslant{a\,b}\Rightarrow{a}\leqslant\sqrt{a\,b}\Rightarrow{b-2\,\sqrt{a\,b}+a}\leqslant{b-a}

(...)




(b -a )2 b-a|b-a|b -a )
|b-a|1/2

Alors f:++
               xx    est 1/ 2 holderienne
(Je n'ai pas appris encore les conditions de Holder mais j'ai vu sur google)
Donc f est uniformement continue sur +???

Posté par
Rana
re : Uniformement continue 19-05-17 à 17:44

Oldboub @ 19-05-2017 à 17:02

Rana,
Dans ton raisonnement, comment as tu fait pour faire apparaître .
Tu cherches à retomber sur la définition...Je pense que tu as considéré en avance que f était uniformément continue...donc ça coince pour ton choix d'epsilon...Donc t'as rien démontré.

On faite d'abord he commence à calculer |f(x)-f(y)|  je fais en sorte qu'elle soit minoré par  constanteconstante puis je la pose plus petite que et là je trouve un delta qui verifie la relation constanteconstante<
Ainsi on trouvera un dependra de et qui verifie la relation |x-y|<|f(x)-f(y)|<.(comme si on a construit pour ce delta trouvé cette relation qu'on a plus besoin de la montré de nouveau car c'est déjà  fait par construction ).

Posté par
jb2017
re : Uniformement continue 19-05-17 à 17:50

Bonjour
@oldboub, bien sûr tu as raison mais implicitement j'exclus le cas  x=y car dans ce cas il n'y a rien à démontrer. De toute façon, ici il s'agit dans les remarques  à arriver à quelque chose qui tienne la route. Ce que simplement je veux pointer  ici, c'est que cette inégalité utilisée par Rana n'est pas le vrai problème. La difficulté est dans l'enchainement des implications où l'on ne s'y retrouve pas.
Finalement @Rana, pour en finir,   je  te corrige mais avant tout je te fais remarquer
que
1. la solution est donnée par  etnopial ( 18-05-17 à 17:15)qui calcule w(f) et cela montrer que x^a est UC sur R^+ pour a<=1 et non UC pour a>1? (nous ci a=1/2).
2. ou par moi (18-05-17 à 18:50)  même si je ne mets pas tous les détails.
3. En fait ce que tu  veux c'est revenir à la définition et c'est normal.  C'est à dire que tu cherches "\delta en fonction de \epsilon  pour obtenir le résultat pour dire les choses simplement"
Selon tes derniers posts il me semble que tu as bien compris la définition et son sens
et tu as commencé a donner une solution pas tout à fait correcte mais on en est pas loin.
Je te donne dans un post à suivre ce vers quoi tu voulais arriver mais il me faut laisser un peu de temps pour rédiger.

Posté par
Rana
re : Uniformement continue 19-05-17 à 18:05

Ok merci et j'attendrais ton post

Posté par
jb2017
re : Uniformement continue 19-05-17 à 18:42

Rebonjour,
Soit 0\leq x< y \in \R^+.  (On ne restreint pas la généralité en supposant que
(x\neq y$ et x plus petit que  y)


première étape (et premier réflexe)Par le théorème des accroissement finis on a:

\exists  c\in ]x,y[,  \frac{f(y)-f(x)}{y-x}=f'(c)=\frac{1}{2 \sqrt{c}}\leq \dfrac{1}{2},
pourvu que 1\leq x<y.

On a donc \forall x,y  \in [1,+\infty[,  |f(x)-f(y)|\leq \frac{1}{2}|x-y|.

On a donc f est 1/2-lipschitzienne sur [1,+\infty[  et  cela implique  (c'est bien connu)  que f est u-c sur  [1,+\infty[.

si ce n'est pas connu c'est facile à voir:

en effet avec \delta=2\epsilon

\forall x,y  \in [1,+\infty[, |x-y|< \delta \Rightarrow  |f(x)-f(y)|< \epsilon.

C'est à peu près ce que tu as écris dans ton dernier post.

deuxième étape: On voit que l'on a un petit problème au voisinage de 0. En effet  
\frac{1}{2 \sqrt{c}} peut être très grand quand x et y sont proches de 0.

Autrement dit f n'est pas k-lipschitzienne sur [0,1]. Si on veut faire une démonstration, c'est plus technique et on devrait en gros reprendre celle qui montre que si f est continue sur  un compact (j'entends ici intervalle fermé borné) elle est uniformément continue sur ce compact.

J'utilise donc ce résultat mais sur [0,2]  (au lieu de [0,1] "pour recoller les morceaux")


troisième étape

On sait que f est uc sur  [0,2] et  sur [1,+\infty[  donc sur    [0,+\infty[.

En effet  on  a  

\forall \epsilon >0,   \exists \delta_1>0,\forall x,y \in [0,2], |x-y|<\delta_1   \Rightarrow   | f(x)-f(y)|<\epsilon


remarque: dans la phrase précédente, on peut toujours remplacer par \delta_1  min(1/2,\[rouge][/rouge]delta_1)

De même

\forall \epsilon >0, \exists \delta_2>0,  \forall  x,y \in [1,+\infty[,  |x-y|<\delta_1   \Rightarrow   | f(x)-f(y)|<\epsilon
  
En choisissant \delta=min(\delta_1,\delta_2,1/2) on a bien

\forall  \epsilon >0,   \exists \delta>0, \forall x,y \in [0,+\infty[, |x-y|<\delta   \Rightarrow   | f(x)-f(y)|<\epsilon

c.q.f.d
  

Posté par
Rana
re : Uniformement continue 19-05-17 à 19:37

Ok merci beaucoup



Vous devez être membre accéder à ce service...

Pas encore inscrit ?

1 compte par personne, multi-compte interdit !

Ou identifiez-vous :


Rester sur la page

Inscription gratuite

Fiches en rapport

parmi 1674 fiches de maths

Désolé, votre version d'Internet Explorer est plus que périmée ! Merci de le mettre à jour ou de télécharger Firefox ou Google Chrome pour utiliser le site. Votre ordinateur vous remerciera !